6
$\begingroup$

Let $x$,$y$,$z$ be positive real numbers which satisfy $xyz=1$. Prove that: $(x^{10}+y^{10}+z^{10})^2 \geq 3(x^{13}+y^{13}+z^{13})$.

And there is a similar question: Let $x$,$y$,$z$ be positive real numbers which satisfy the inequality $(2x^4+3y^4)(2y^4+3z^4)(2z^4+3x^4) \leq(3x+2y)(3y+2z)(3z+2x)$. Prove this inequality: $xyz\leq 1$.

$\endgroup$
4
  • 11
    $\begingroup$ Can you tell us why you want to prove these inequalities and how they arose? $\endgroup$
    – David Roberts
    Mar 9, 2021 at 9:57
  • $\begingroup$ Also, maybe wait a little bit between questions. You have asked two in quick succession. $\endgroup$
    – David Roberts
    Mar 9, 2021 at 9:58
  • $\begingroup$ thank you for your question,I asked the two question just out of curosity, neither for examination nor profit propose.I met this two questions online,here is the link, and I translate it to English. mp.weixin.qq.com/s/w9_isyUk5ie0Oo69c7g9lg $\endgroup$ Mar 9, 2021 at 11:51
  • 3
    $\begingroup$ OK, fair enough. It is good to give the source, if the question is not original to you. :-) $\endgroup$
    – David Roberts
    Mar 9, 2021 at 21:10

8 Answers 8

9
$\begingroup$

These inequalities are algebraic and thus can be proved purely algorithmically.

Mathematica takes a minute or two for this proof of your first inequality:

notebook 1


Here is a "more human" proof:

Substituting $z=\frac1{xy}$, rewrite your first inequality as \begin{equation} f(x,y)\mathrel{:=}\left(\frac{1}{x^{10} y^{10}}+x^{10}+y^{10}\right)^2-3 \left(\frac{1}{x^{13} y^{13}}+x^{13}+y^{13}\right)\ge0 \end{equation} and then as \begin{align} g(x,y)&\mathrel{:=}f(x,y)x^{20} y^{20} \\ &=x^{40} y^{20}-3 x^{33} y^{20}+2 x^{30} y^{30}+x^{20} y^{40}-3 x^{20} y^{33} \\ &+2 x^{20} y^{10}+2 x^{10} y^{20}-3 x^7 y^7+1\ge0, \end{align} for $x,y>0$.

Further, \begin{align} g_1(x,y)\mathrel{:=}{}\frac{g'_x(x,y)}{x^6 y^7}&=40 x^{33} y^{13}-99 x^{26} y^{13}+60 x^{23} y^{23}+20 x^{13} y^{33} \\ &-60 x^{13} y^{26}+40 x^{13} y^3+20 x^3 y^{13}-21, \\ g_2(x,y)\mathrel{:=}\frac{g'_y(x,y)}{x^7 y^6}&=20 x^{33} y^{13}-60 x^{26} y^{13}+60 x^{23} y^{23}+40 x^{13} y^{33} \\ &-99 x^{13} y^{26}+20 x^{13} y^3+40 x^3 y^{13}-21, \end{align} and the only positive roots of the resultants of $g_1(x,y)$ and $g_2(x,y)$ with respect to $x$ and $y$ are $y=1$ and $x=1$, respectively. So, $(1,1)$ is the only critical point of $g$.

Next, all the coefficients of the polynomial $g(1+u,1+v)$ in $u,v$ are nonnegative. Therefore and because of the symmetry $x\leftrightarrow y$, it remains to consider the cases (i) $0\le x\le1$ and $y>0$ is large enough and (ii) $x=0$.

For case (i), we have $g(x,y)\ge1 - 3 x^7 y^7 + 2 x^{10} y^{20}>0$. For case (ii), we have $g(0,y)=1>0$.

So, your first inequality is proved, again.


It took Mathematica about 1.8 hours to prove your second inequality (click on the image to enlarge it):

notebook 2

The latter proof would probably take many thousands pages.

$\endgroup$
5
  • 1
    $\begingroup$ What does "$y = \infty-$" mean? $\endgroup$
    – LSpice
    Mar 9, 2021 at 15:49
  • 1
    $\begingroup$ @LSpice : I have replaced that by "$y>0$ is large enough". $\endgroup$ Mar 9, 2021 at 16:02
  • 1
    $\begingroup$ Thanks! Just so that it's here in fulltext: the first Mathematica command is Reduce[(x^(10)+y^(10)+z^(10))^2<3(x^(13)+y^(13)+z^(13))&&x>0&&y>0&&z>0&&x y z>1,{x,y,z},Reals], and the second is Reduce[(2x^4+3y^4)(3x^4+2z^4)(2y^4+3z^4)<=(3x+2y)(3y+2z)(2z+3z)&&x>0&&y>0&&z>0&&x y z>1,{x,y,z},Reals]//AbsoluteTiming. Since the inequalities seem to be true (at least, you prove the first one), what does it mean that Mathematica returns False? $\endgroup$
    – LSpice
    Mar 9, 2021 at 21:51
  • 2
    $\begingroup$ @LSpice : Both "inequalities" are actually implications of the form $C:=(A\implies B)$. In both cases, I asked Mathematica to Reduce[] the negation $\neg C=(A\ \&\ \neg B)$ of the implication $C$ (where $\neg$ is the negation symbol, $A$ is an equality or an inequality, and $B$ is an inequality). In both cases, Mathematica Reduce[]'d the negation $\neg C$ of $C$ to False. So, $C$ is true. $\endgroup$ Mar 10, 2021 at 1:17
  • $\begingroup$ thank you for your detailed solution and explaination. the application of calculus is awesome.it's so amzing to use Mathematica to prove or disprove inequality.Never have I realised that this software can be so powerful. $\endgroup$ Mar 10, 2021 at 2:30
7
$\begingroup$

Here is a little less computer-assisted approach to both inequalities than the one suggested by Iosif Pinelis. Namely, I use the properties of rational one-variable functions which are seen from their graphs without thinking about how to prove them rigorously.

  1. Fix $xyz=1$ and $x^{10}+y^{10}+z^{10}:=S$. Look for a maximum of $x^{13}+y^{13}+z^{13}$. It is achieved (the set of admissible triples is compact), and at the maximum points the gradients of $xyz,x^{10}+y^{10}+z^{10},x^{13}+y^{13}+z^{13}$ are linearly dependent, that is, we should have $$\alpha\cdot(yz,xz,xy)+\beta\cdot 10(x^9,y^9,z^9)+\gamma\cdot 13(x^{12},y^{12},z^{12})=0$$ where the real coefficients $(\alpha,\beta,\gamma)$ are not simultaneously zero. This means that all numbers $x,y,z$ solve the same equation $f(t):=a+bt^{10}+ct^{13}=0$, where $a=\alpha xyz$, $b=10\beta$, $c=13\gamma$. Such an equation may have at most two different positive solutions by Descartes' rule of signs. That is, two of $x,y,z$ must be equal. Without loss of generality we may assume that $y=x$, then $z=1/x^2$ and we should prove a 1-variable inequality $$(2x^{10}+x^{-20})^2\geqslant 3(2x^{13}+x^{-26}).$$ I do not see any nice explanation why this is true, but looking at the graphs we see that the ratio of exponents 13:10 may be increased to approximately 1.4047 (you may see that this graph is above the x-axis, but for the value of parameter 1.4048 it crosses it already.

  2. Denote $a=y/x,b=z/y,c=x/z$. Then $abc=1$ and we are given $(xyz)^4(2+3a^4)(2+3b^4)(2+3c^4)\leqslant xyz(3+2a)(3+2b)(3+2c)$. Thus for establishing $xyz\leqslant 1$ it suffices (and is actually necessary) to check that $$(3+2a)(3+2b)(3+2c)\leqslant (2+3a^4)(2+3b^4)(2+3c^4) \quad (1)$$ whenever $a,b,c$ are positive numbers with $abc=1$. (1) is equivalent to $$ F(a,b,c):=h(a)+h(b)+h(c)\geqslant 0, \,\, \text{where}\,\, h(x):=\log(2+3x^4)-\log(3+2x). $$ First of all, I claim that $F$ attains its minimal value on the set $\Omega:=\{(a,b,c):abc=1, a,b,c>0\}$. Indeed, consider a sequence $(a_n,b_n,c_n)$ for which $F(a_n,b_n,c_n)$ approaches the infimum of $F$ on $\Omega$. Since $h(x)$ is bounded from below on $(0,\infty)$ and tends to $+\infty$ for large $x$, we conclude that $a_n,b_n,c_n$ must be bounded, then we may choose a convergent subsequence and get a minimizer. So, denote the minimizer by $(a_0,b_0,c_0)$. the gradients of $F(a,b,c)$ and $abc$ at the point $(a_0,b_0,c_0)$ must be linearly dependent, thus we may write $h'(a_0)=\lambda b_0c_0$, $h'(b_0)=\lambda a_0c_0$, $h'(c_0)=\lambda b_0a_0$ for certain real $\lambda$. In other words, the function $g(x):=xh'(x)$ takes the same value at points $a_0,b_0,c_0$. Looking at the plot of $g(x)$ for positive $x$ we see that it takes each value at most twice. Thus two of three variables $a_0,b_0,c_0$ must be equal. Without loss of generality $a_0=b_0=:x$, $c_0=1/x^2$, and we should prove a 1-variable polynomial inequality $$(2+3x^4)^2(2+3/x^8)\geqslant (3+2x)^2(3+2/x^2)\quad \text{for}\quad x>0.$$ Well, it follows from factorization.

$\endgroup$
4
$\begingroup$

Here is a simple proof:

By Chebyshev sum inequality and AM-GM, we have \begin{align*} x^{13}x^{1/3} + y^{13}y^{1/3} + z^{13}z^{1/3} &\ge \frac13(x^{13} + y^{13} + z^{13})(x^{1/3} + y^{1/3} + z^{1/3})\\ &\ge \frac13(x^{13} + y^{13} + z^{13})\cdot 3\sqrt[3]{x^{1/3}y^{1/3}z^{1/3}}\\ &= x^{13} + y^{13} + z^{13}. \tag{1} \end{align*}

Then, it suffices to prove that $$(x^{10}+y^{10}+z^{10})^2 \geq 3(x^{13+\frac13}+y^{13 + \frac13}+z^{13+ \frac13}). \tag{2}$$

Letting $x = a^{3/10}, y = b^{3/10}, z = c^{3/10}$, it suffices to prove that, for all $a, b, c > 0$ with $abc = 1$, $$(a^3 + b^3 + c^3)^2 \ge 3(a^4 + b^4 + c^4). \tag{3}$$

We have \begin{align*} &(a^3 + b^3 + c^3)^2 - 3(a^4 + b^4 + c^4)\\ ={}& a^6 + b^6 + c^6 + 2a^3b^3 + 2b^3c^3 + 2c^3a^3 - 3(a^4 + b^4 + c^4)\\ \ge{}& a^6 + b^6 + c^6 + (3a^2b^2 - 1) + (3b^2c^2 - 1) + (3c^2a^2 - 1) - 3(a^4 + b^4 + c^4) \tag{4}\\ ={}& a^6 + b^6 + c^6 + 3a^2b^2 + 3b^2c^2 + 3c^2a^2 - 3a^4 - 3b^4 - 3c^4 - 3\\ ={}& a^6 + b^6 + c^6 + 3a^2b^2 + 3b^2c^2 + 3c^2a^2 - 3a^4 - 3b^4 - 3c^4 - 3a^2b^2c^2\\ ={}& (a^2 + b^2 + c^2 - 3)(a^4 + b^4 + c^4 -a^2b^2 - b^2c^2 - c^2a^2)\\ \ge{}& 0 \end{align*} where we use $2a^3b^3 - (3a^2b^2 - 1) = (2ab + 1)(ab - 1)^2 \ge 0$ in (4).

We are done.

$\endgroup$
3
$\begingroup$

For the first inequality, let us write $x=X^3$, $y=Y^3$, $z=Z^3$, and pass to a homogenized version of it, obtained by multiplying the R.H.S. with $(xyz)^{7/3}=(XYZ)^7$. So we have to show: $$ (X^{30}+Y^{30}+Z^{30})^2 \ge 3 (X^{39}+Y^{39}+Z^{39})X^7Y^7Z^7\ . $$ Now divide by $Z^{60}$ to dehomogenize, so it is enough to show the above for $Z=1$ and arbitrary $X,Y\ge 0$. Consider the difference function $$ f(X,Y)= (X^{30}+Y^{30}+1)^2 - 3 (X^{39}+Y^{39}+1)X^7Y^7\ . $$ Since $f(1,1)=0$ and on the boundary $XY=0$ we have $f(X,Y)\ge 1$, a global minimal value is a local minimum in the domain $Y,Y>0$. We search such local minima, they are among the solutions of $f'(X,Y)=0$, which leads to the algebraic system of equations $$ \left\{ \begin{aligned} 2(X^{30}+Y^{30}+1)\cdot 30X^{29} &= 3 (46X^{45}Y^7 + 7X^6Y^{46}+7X^6Y^7)\ ,\\ 2(X^{30}+Y^{30}+1)\cdot 30Y^{29} &= 3 (46Y^{45}X^7 + 7Y^6X^{46}+7Y^6X^7)\ . \end{aligned} \right. $$ Now multiply the first equation with $X$, the second with $Y$. This leads to $$ 2(X^{30}+Y^{30}+1)\cdot 30X^{30} = 3 (46X^{46}Y^7 + 7X^7Y^{46}+7X^7Y^7) = 2(X^{30}+Y^{30}+1)\cdot 30Y^{30} \ . $$ So a solution must satisfy $X=Y$. We plug in this into the above, get $$ 2(2X^{30}+1)\cdot 30X^{30} = 3 (53X^{53}+7X^{14}) \ . $$ It turns out that $X=1$ is the only real positive root. So $(X,Y)=(1,1)$ is the only critical point. So it is the point where $f$ is globaly minimal.


The second point follows from the inequality $$ (2x^4+3y^4)(2y^4+3z^4)(2z^4+3x^4) \ge (3x+2y)(3y+2z)(3z+2x)x^3y^3z^3\ . $$ Let us show the above. After we expand, it is enough to show the domination $$ 12 \, \sum x^{8} y^{4} + 18 \sum \, y^{8} x^{4} \ge 18 \, \sum x^{5} y^{4} z^{3} + 12 \sum\, x^{5} y^{3} z^{4} \ . $$ To have a better view, let us place the points $(i,j,k)$ corresponding to the monomials $x^iy^jz^k$ that occur in the plane $i+j+k=12$.

mathoverflow 385942

Now, a simple human scheme of domination can be found, for instance exploiting: $$ \begin{aligned} 12(5,3,4) &= 5(8,4,0)+ 2(0,8,4) + 5(4,0,8)\ ,\\ 12(5,4,3) &= 5(4,8,0)+ 2(0,4,8) + 5(8,0,4)\ . \end{aligned} $$ Now using the Jensen-convexity of the logarithm in the form $au+bv+cw\ge u^av^bw^c$ for $u,v,w>0$ and weights $a,b,c>0$ with total sum $a+b+c=1$, for the above suggested weights $a=c=5/12$, $b=2/12$, and the monomials $u=x^8y^4$, $v=y^8z^4$, $w=z^8x^4$ for the first line, then the reversed version for the second line, gives $$ \begin{aligned} 12\,x^5y^3z^4 &\le 5\,x^8y^4 + 2\,y^8z^4 + 5\,z^8x^4\ ,\\ 12\,x^5y^4z^3 &\le 5\,x^4y^8 + 2\,y^4z^8 + 5\,z^4x^8\ . \end{aligned} $$

This concludes the needed domination.

$\endgroup$
1
  • $\begingroup$ The second one is nice. $\endgroup$
    – River Li
    Oct 16, 2022 at 1:58
3
$\begingroup$

Another way. By my previous post it's enough to prove that: $$(x^5+y^5+z^5)^2\geq3xyz(x^7+y^7+z^7)$$ for positive $x$, $y$ and $z$.

Indeed, let $x^5+y^5+z^5=\text{constant}$ and $x^7+y^7+z^7=\text{constant}$.

Thus, by the Vasc's EV Method (see here: Cîrtoaje - The equal variable method Corollary 1.8(b)) it's enough to prove the last inequality (because it's homogeneous) for $y=z=1$, which gives $$(x^5+2)^2\geq3x(x^7+2)$$ or $$(x-1)^2(x^8+2x^7-2x^5-4x^4-2x^3+2x+4)\geq0$$ and the rest is smooth.

$\endgroup$
2
$\begingroup$

This is to complement the nice answer by Fedor Petrov by a calculus proof of
the inequality $$ (2 x^{10} + x^{-20})^2\ge3 (2 x^{13} + x^{-26}),$$ for real $x>0$.

Rewrite this inequality as $$f(x):=4 x^{60}-6 x^{53}+4 x^{30}-3 x^{14}+1\ge0.$$ Let $$f_1(x):=\frac{f'(x)}{6x^{13}}=40 x^{46}-53 x^{39}+20 x^{16}-7,\\f_2(x):=\frac{f_1'(x)}{x^{15}}=1840 x^{30}-2067 x^{23}+320,\\ f_3(x):=\frac{f_2'(x)}{69x^{22}}=800 x^7-689.$$ Then, clearly, $f_2$ attains its minimum (on $[0,\infty)$) at $(689/800)^{1/7}$, and this minimum is $24.7\ldots>0$. So, $f_2>0$ and hence $f_1$ is increasing, from $f_1(0)=-7<0$ to $f_1(\infty-)=\infty>0$. Also, $f_1(1)=0$. So, $f_1\le0$ on $[0,1]$ and $f_1\ge0$ on $[1,\infty)$. So, $f$ attains its minimum (on $[0,\infty)$) at $1$, and this minimum is $0$.

$\endgroup$
1
$\begingroup$

We'll prove a stronger inequality:

$$\left(a^{10}+b^{10}+c^{10}\right)^2\geq3(a^{14}+b^{14}+c^{14}),$$ where $abc=1,$

for which it's enough to prove that $$(x^5+y^5+z^5)^2\geq3xyz(x^7+y^7+z^7)$$ for positives $x$, $y$ and $z$.

Indeed, let $x=\min\{x,y,z\}$, $y=x+u$ and $z=x+v$.

Hence, $$\left(x^5 + y^5 + z^5\right)^2-3xyz\left(x^7 +y^7 + z^7\right)=$$ $$=(u^2-uv+v^2)x^8+8(-u^3+2u^2v+2uv^2-v^3)x^7+$$ $$+4(5u^4-17u^3v+50u^2v^2-17uv^3+5v^4)x^6+$$ $$+8(11u^5-20u^4v+25u^2v^2+25u^2v^3-20uv^4+11v^5)x^5+$$ $$+2(63u^6-79u^5v+50u^4v^2+100u^3v^3+50u^2v^4-79uv^5+63u^6)x^4+$$ $$+4(24u^7-21u^6v+5u^5v^2+25u^4v^3+25u^3v^4+5u^2v^5-21uv^6+24v^7)x^3+$$ $$+2(21u^8-12u^7v+10u^5v^3+25u^4v^4+10u^3v^5-12uv^7+21v^8)x^2+$$ $$+(10u^9-3u^8v+10u^5v^4+10u^4v^5-3uv^8+10v^9)x+(u^5+v^5)^2\geq$$ $$\geq\left((u^2-uv+v^2)x^2-8(u+v)(u^2-3uv+v^2)x+4(5u^4-17u^3v+50u^2v^2-17uv^3+5v^4)\right)x^6.$$

Let $u^2+v^2=tuv$.

Hence, $t\geq2$ and it remains to prove that $$(t-1)(5t^2-17t+40)-4(t+2)(t-3)^2\geq0,$$ which is true because $$(t-1)(5t^2-17t+40)-4(t+2)(t-3)^2=$$ $$=t^3-6t^2+69t-112=t(t-3)^2+60t-112\geq0.$$ Id est, it's enough to prove that $$x^{14}+y^{14}+z^{14}\geq x^{13}+y^{13}+z^{13},$$ where $x$, $y$ and $z$ are positives such that $xyz=1$, or $$x^{42}+y^{42}+z^{42}\geq (x^{39}+y^{39}+z^{39})xyz,$$ which is true by Muirhed because $$(42,0,0)\succ(40,1,1).$$

$\endgroup$
1
$\begingroup$

This is to give an alternative proof of the inequality in Fedor Petrov's nice answer $$(2 x^{10} + x^{-20})^2\ge3 (2 x^{13} + x^{-26})$$ for all $x > 0$.

We have \begin{align*} &2x^{13+1/3} + x^{-26-2/3} - (2x^{13} + x^{-26})\\ ={}& 2(x^{13+1/3} - x^{13}) + (x^{-13-1/3} - x^{-13}) (x^{-13-1/3} + x^{-13})\\ ={}& x^{13}(x^{1/3}-1)(2 - x^{-119/3} - x^{-118/3})\\ \ge{}& 0. \end{align*}

Thus, it suffices to prove that $$(2 x^{10} + x^{-20})^2\ge 3(2x^{13+1/3} + x^{-26-2/3}).$$

Letting $x = a^{3/10}$, it suffices to prove that, for all $a > 0$, $$(2a^3 + a^{-6})^2 \ge 3(2a^4 + a^{-8}).$$

We have \begin{align*} &(2a^3 + a^{-6})^2 - 3(2a^4 + a^{-8})\\ ={}& 4a^6 + 4a^{-3}+ a^{-12} - 6a^4 - 3a^{-8}\\ \ge\,& 4a^6 + 2(3a^{-2} - 1) + a^{-12} - 6a^4 - 3a^{-8}\\ ={}& \frac{(2a^2+1)(2a^{12} - 2a^6 + 1)(a^2-1)^2}{a^{12}}\\ \ge{}& 0 \end{align*} where we have used $2a^{-3} - (3a^{-2} - 1) = \frac{(a+2)(a-1)^2}{a^3} \ge 0$.

We are done.

$\endgroup$
2
  • $\begingroup$ TeX note: in an {align} environment, as you know, = & … at the beginning of a line results in no space after the =. You manually inserted a space \,, but it is probably preferable to insert an empty atom {} after the = which tricks it into spacing correctly itself. While adding a link to @FedorPetrov's answer that you reference, I edited accordingly. $\endgroup$
    – LSpice
    Jan 11, 2023 at 2:53
  • 1
    $\begingroup$ @LSpice Thank you! $\endgroup$
    – River Li
    Jan 11, 2023 at 2:57

Your Answer

By clicking “Post Your Answer”, you agree to our terms of service and acknowledge you have read our privacy policy.

Not the answer you're looking for? Browse other questions tagged or ask your own question.